[Arm-netbook] physics analysis needed of belt-driven pulley system

Luke Kenneth Casson Leighton lkcl at lkcl.net
Sat Aug 19 10:51:01 BST 2017


some background:
http://forums.reprap.org/read.php?177,767087,784083#msg-784083

and the same question here:
https://physics.stackexchange.com/questions/352661/

can anyone help?  basically - and i really should have done this
before going ahead... *sigh*... - when you have a pulley system is the
amount of force on the belt less, equal, or greater, under the same
*print-head* acceleration conditions, compared to a *non* pulley
system?

l.



More information about the arm-netbook mailing list